Difference between revisions of "2008 AMC 10A Problems/Problem 3"

m (Problem)
Line 9: Line 9:
 
==See also==
 
==See also==
 
{{AMC10 box|year=2008|ab=A|num-b=2|num-a=4}}
 
{{AMC10 box|year=2008|ab=A|num-b=2|num-a=4}}
 +
{{MAA Notice}}

Revision as of 11:52, 4 July 2013

Problem

For the positive integer $n$, let $<n>$ denote the sum of all the positive divisors of $n$ with the exception of $n$ itself. For example, $<4>=1+2=3$ and $<12>=1+2+3+4+6=16$. What is $<<<6>>>$?

$\mathrm{(A)}\ 6\qquad\mathrm{(B)}\ 12\qquad\mathrm{(C)}\ 24\qquad\mathrm{(D)}\ 32\qquad\mathrm{(E)}\ 36$

Solution

$<<<6>>>\ =\ <<6>>\ =\ <6>\ =\ 6\quad\Longrightarrow\quad\mathrm{(A)}$

See also

2008 AMC 10A (ProblemsAnswer KeyResources)
Preceded by
Problem 2
Followed by
Problem 4
1 2 3 4 5 6 7 8 9 10 11 12 13 14 15 16 17 18 19 20 21 22 23 24 25
All AMC 10 Problems and Solutions

The problems on this page are copyrighted by the Mathematical Association of America's American Mathematics Competitions. AMC logo.png